Determination of the order of a pole












0














In the function $$f(z) =frac{sin(frac{pi}{2}(z+1))}{(z^2+2z+4)(z+1)^3}$$ the order of the pole in $z=-1$ is correctly two?
Or maybe it is an eliminable singularity?



I have a problem because often if I have an eliminable singularity I only have to Taylor expand the numerator of the fraction at the first term, in this situation, if I expand at the first term I have a singularity of order two and if expand at the second it disappears without becoming zero.










share|cite|improve this question
























  • Yes, it is two. If you take the limit when $;zto-1;$ of the numerator with $;(z+1);$ in the denominator, the limit is finite , so the remaining exponent in the denominator of $;z+1;$ is the order of the pole at $;z=-1;$
    – DonAntonio
    Nov 27 '18 at 21:40












  • All you need to do is to plug the expansions $$sin u=uleft(1-frac{u^2}6+o(u^2)right)$$ and $$frac1{u^2+3}=frac13left(1-frac13u^2+o(u^2)right)$$ when $uto0$, in the fraction and, using the shorthand $w=z+1$, to deduce the correct expansion $$f(z)=frac{fracpi2w}{w^3}frac13left(1-frac{pi^2w^2}{24}+o(w^2)right)left(1-frac{w^2}3+o(w^3)right)$$ that is $$f(z)=fracpi{6w^2}left(1-cw^2+o(w^2)right)=fracpi{6w^2}-cfracpi6+o(1)$$ for some suitable $cne0$, indicating a pole of order $2$ and a residue $0$ at $z=-1$.
    – Did
    Nov 27 '18 at 21:50
















0














In the function $$f(z) =frac{sin(frac{pi}{2}(z+1))}{(z^2+2z+4)(z+1)^3}$$ the order of the pole in $z=-1$ is correctly two?
Or maybe it is an eliminable singularity?



I have a problem because often if I have an eliminable singularity I only have to Taylor expand the numerator of the fraction at the first term, in this situation, if I expand at the first term I have a singularity of order two and if expand at the second it disappears without becoming zero.










share|cite|improve this question
























  • Yes, it is two. If you take the limit when $;zto-1;$ of the numerator with $;(z+1);$ in the denominator, the limit is finite , so the remaining exponent in the denominator of $;z+1;$ is the order of the pole at $;z=-1;$
    – DonAntonio
    Nov 27 '18 at 21:40












  • All you need to do is to plug the expansions $$sin u=uleft(1-frac{u^2}6+o(u^2)right)$$ and $$frac1{u^2+3}=frac13left(1-frac13u^2+o(u^2)right)$$ when $uto0$, in the fraction and, using the shorthand $w=z+1$, to deduce the correct expansion $$f(z)=frac{fracpi2w}{w^3}frac13left(1-frac{pi^2w^2}{24}+o(w^2)right)left(1-frac{w^2}3+o(w^3)right)$$ that is $$f(z)=fracpi{6w^2}left(1-cw^2+o(w^2)right)=fracpi{6w^2}-cfracpi6+o(1)$$ for some suitable $cne0$, indicating a pole of order $2$ and a residue $0$ at $z=-1$.
    – Did
    Nov 27 '18 at 21:50














0












0








0







In the function $$f(z) =frac{sin(frac{pi}{2}(z+1))}{(z^2+2z+4)(z+1)^3}$$ the order of the pole in $z=-1$ is correctly two?
Or maybe it is an eliminable singularity?



I have a problem because often if I have an eliminable singularity I only have to Taylor expand the numerator of the fraction at the first term, in this situation, if I expand at the first term I have a singularity of order two and if expand at the second it disappears without becoming zero.










share|cite|improve this question















In the function $$f(z) =frac{sin(frac{pi}{2}(z+1))}{(z^2+2z+4)(z+1)^3}$$ the order of the pole in $z=-1$ is correctly two?
Or maybe it is an eliminable singularity?



I have a problem because often if I have an eliminable singularity I only have to Taylor expand the numerator of the fraction at the first term, in this situation, if I expand at the first term I have a singularity of order two and if expand at the second it disappears without becoming zero.







complex-analysis taylor-expansion singularity






share|cite|improve this question















share|cite|improve this question













share|cite|improve this question




share|cite|improve this question








edited Nov 27 '18 at 21:51









Did

246k23220454




246k23220454










asked Nov 27 '18 at 21:35









pter26

307111




307111












  • Yes, it is two. If you take the limit when $;zto-1;$ of the numerator with $;(z+1);$ in the denominator, the limit is finite , so the remaining exponent in the denominator of $;z+1;$ is the order of the pole at $;z=-1;$
    – DonAntonio
    Nov 27 '18 at 21:40












  • All you need to do is to plug the expansions $$sin u=uleft(1-frac{u^2}6+o(u^2)right)$$ and $$frac1{u^2+3}=frac13left(1-frac13u^2+o(u^2)right)$$ when $uto0$, in the fraction and, using the shorthand $w=z+1$, to deduce the correct expansion $$f(z)=frac{fracpi2w}{w^3}frac13left(1-frac{pi^2w^2}{24}+o(w^2)right)left(1-frac{w^2}3+o(w^3)right)$$ that is $$f(z)=fracpi{6w^2}left(1-cw^2+o(w^2)right)=fracpi{6w^2}-cfracpi6+o(1)$$ for some suitable $cne0$, indicating a pole of order $2$ and a residue $0$ at $z=-1$.
    – Did
    Nov 27 '18 at 21:50


















  • Yes, it is two. If you take the limit when $;zto-1;$ of the numerator with $;(z+1);$ in the denominator, the limit is finite , so the remaining exponent in the denominator of $;z+1;$ is the order of the pole at $;z=-1;$
    – DonAntonio
    Nov 27 '18 at 21:40












  • All you need to do is to plug the expansions $$sin u=uleft(1-frac{u^2}6+o(u^2)right)$$ and $$frac1{u^2+3}=frac13left(1-frac13u^2+o(u^2)right)$$ when $uto0$, in the fraction and, using the shorthand $w=z+1$, to deduce the correct expansion $$f(z)=frac{fracpi2w}{w^3}frac13left(1-frac{pi^2w^2}{24}+o(w^2)right)left(1-frac{w^2}3+o(w^3)right)$$ that is $$f(z)=fracpi{6w^2}left(1-cw^2+o(w^2)right)=fracpi{6w^2}-cfracpi6+o(1)$$ for some suitable $cne0$, indicating a pole of order $2$ and a residue $0$ at $z=-1$.
    – Did
    Nov 27 '18 at 21:50
















Yes, it is two. If you take the limit when $;zto-1;$ of the numerator with $;(z+1);$ in the denominator, the limit is finite , so the remaining exponent in the denominator of $;z+1;$ is the order of the pole at $;z=-1;$
– DonAntonio
Nov 27 '18 at 21:40






Yes, it is two. If you take the limit when $;zto-1;$ of the numerator with $;(z+1);$ in the denominator, the limit is finite , so the remaining exponent in the denominator of $;z+1;$ is the order of the pole at $;z=-1;$
– DonAntonio
Nov 27 '18 at 21:40














All you need to do is to plug the expansions $$sin u=uleft(1-frac{u^2}6+o(u^2)right)$$ and $$frac1{u^2+3}=frac13left(1-frac13u^2+o(u^2)right)$$ when $uto0$, in the fraction and, using the shorthand $w=z+1$, to deduce the correct expansion $$f(z)=frac{fracpi2w}{w^3}frac13left(1-frac{pi^2w^2}{24}+o(w^2)right)left(1-frac{w^2}3+o(w^3)right)$$ that is $$f(z)=fracpi{6w^2}left(1-cw^2+o(w^2)right)=fracpi{6w^2}-cfracpi6+o(1)$$ for some suitable $cne0$, indicating a pole of order $2$ and a residue $0$ at $z=-1$.
– Did
Nov 27 '18 at 21:50




All you need to do is to plug the expansions $$sin u=uleft(1-frac{u^2}6+o(u^2)right)$$ and $$frac1{u^2+3}=frac13left(1-frac13u^2+o(u^2)right)$$ when $uto0$, in the fraction and, using the shorthand $w=z+1$, to deduce the correct expansion $$f(z)=frac{fracpi2w}{w^3}frac13left(1-frac{pi^2w^2}{24}+o(w^2)right)left(1-frac{w^2}3+o(w^3)right)$$ that is $$f(z)=fracpi{6w^2}left(1-cw^2+o(w^2)right)=fracpi{6w^2}-cfracpi6+o(1)$$ for some suitable $cne0$, indicating a pole of order $2$ and a residue $0$ at $z=-1$.
– Did
Nov 27 '18 at 21:50










1 Answer
1






active

oldest

votes


















1














We have



$$frac{sinfracpi2(z+1)}{left[3+(z+1)^2right](z+1)^3}=frac13cdotfrac1{1+left(frac{z+1}{sqrt3}right)^2}cdotfrac1{(z+1)^3}sum_{n=0}^inftyfrac{(-1)^npi^{2n+1}(z+1)^{2n+1}}{(2n+1)!}=$$



$$=frac13left(1-left(frac{z+1}{sqrt3}right)^2+left(frac{z+1}{sqrt3}right)^4-ldotsright)frac1{(z+1)^{color{red}2}}sum_{n=0}^inftyfrac{(-1)^npi^{2n+1}(z+1)^{color{red}{2n}}}{(2n+1)!}$$



and we can see clearly in the above Laurent expansion that the pole at $;z=-1;$ is of order two. Of course, the above is valid only in some neighbourhood of $;-1;$.






share|cite|improve this answer























  • Why is the first equality possible?
    – pter26
    Nov 27 '18 at 21:54










  • @pter26 That's merely algebra and the Taylor expansion of sine, which is true for any value of $;zinBbb C;$ .
    – DonAntonio
    Nov 27 '18 at 21:55










  • Thanks a lot! So can I tell the residue in $z=-1$ is one from this?
    – pter26
    Nov 27 '18 at 22:02












  • Well, I think it is zero since in the series the first non-constant element goes with $;(z+1)^2;$ , so " there is no coefficient of $;(z+1)^{-1};$" , meaning: its coefficient is zero
    – DonAntonio
    Nov 27 '18 at 22:15











Your Answer





StackExchange.ifUsing("editor", function () {
return StackExchange.using("mathjaxEditing", function () {
StackExchange.MarkdownEditor.creationCallbacks.add(function (editor, postfix) {
StackExchange.mathjaxEditing.prepareWmdForMathJax(editor, postfix, [["$", "$"], ["\\(","\\)"]]);
});
});
}, "mathjax-editing");

StackExchange.ready(function() {
var channelOptions = {
tags: "".split(" "),
id: "69"
};
initTagRenderer("".split(" "), "".split(" "), channelOptions);

StackExchange.using("externalEditor", function() {
// Have to fire editor after snippets, if snippets enabled
if (StackExchange.settings.snippets.snippetsEnabled) {
StackExchange.using("snippets", function() {
createEditor();
});
}
else {
createEditor();
}
});

function createEditor() {
StackExchange.prepareEditor({
heartbeatType: 'answer',
autoActivateHeartbeat: false,
convertImagesToLinks: true,
noModals: true,
showLowRepImageUploadWarning: true,
reputationToPostImages: 10,
bindNavPrevention: true,
postfix: "",
imageUploader: {
brandingHtml: "Powered by u003ca class="icon-imgur-white" href="https://imgur.com/"u003eu003c/au003e",
contentPolicyHtml: "User contributions licensed under u003ca href="https://creativecommons.org/licenses/by-sa/3.0/"u003ecc by-sa 3.0 with attribution requiredu003c/au003e u003ca href="https://stackoverflow.com/legal/content-policy"u003e(content policy)u003c/au003e",
allowUrls: true
},
noCode: true, onDemand: true,
discardSelector: ".discard-answer"
,immediatelyShowMarkdownHelp:true
});


}
});














draft saved

draft discarded


















StackExchange.ready(
function () {
StackExchange.openid.initPostLogin('.new-post-login', 'https%3a%2f%2fmath.stackexchange.com%2fquestions%2f3016333%2fdetermination-of-the-order-of-a-pole%23new-answer', 'question_page');
}
);

Post as a guest















Required, but never shown

























1 Answer
1






active

oldest

votes








1 Answer
1






active

oldest

votes









active

oldest

votes






active

oldest

votes









1














We have



$$frac{sinfracpi2(z+1)}{left[3+(z+1)^2right](z+1)^3}=frac13cdotfrac1{1+left(frac{z+1}{sqrt3}right)^2}cdotfrac1{(z+1)^3}sum_{n=0}^inftyfrac{(-1)^npi^{2n+1}(z+1)^{2n+1}}{(2n+1)!}=$$



$$=frac13left(1-left(frac{z+1}{sqrt3}right)^2+left(frac{z+1}{sqrt3}right)^4-ldotsright)frac1{(z+1)^{color{red}2}}sum_{n=0}^inftyfrac{(-1)^npi^{2n+1}(z+1)^{color{red}{2n}}}{(2n+1)!}$$



and we can see clearly in the above Laurent expansion that the pole at $;z=-1;$ is of order two. Of course, the above is valid only in some neighbourhood of $;-1;$.






share|cite|improve this answer























  • Why is the first equality possible?
    – pter26
    Nov 27 '18 at 21:54










  • @pter26 That's merely algebra and the Taylor expansion of sine, which is true for any value of $;zinBbb C;$ .
    – DonAntonio
    Nov 27 '18 at 21:55










  • Thanks a lot! So can I tell the residue in $z=-1$ is one from this?
    – pter26
    Nov 27 '18 at 22:02












  • Well, I think it is zero since in the series the first non-constant element goes with $;(z+1)^2;$ , so " there is no coefficient of $;(z+1)^{-1};$" , meaning: its coefficient is zero
    – DonAntonio
    Nov 27 '18 at 22:15
















1














We have



$$frac{sinfracpi2(z+1)}{left[3+(z+1)^2right](z+1)^3}=frac13cdotfrac1{1+left(frac{z+1}{sqrt3}right)^2}cdotfrac1{(z+1)^3}sum_{n=0}^inftyfrac{(-1)^npi^{2n+1}(z+1)^{2n+1}}{(2n+1)!}=$$



$$=frac13left(1-left(frac{z+1}{sqrt3}right)^2+left(frac{z+1}{sqrt3}right)^4-ldotsright)frac1{(z+1)^{color{red}2}}sum_{n=0}^inftyfrac{(-1)^npi^{2n+1}(z+1)^{color{red}{2n}}}{(2n+1)!}$$



and we can see clearly in the above Laurent expansion that the pole at $;z=-1;$ is of order two. Of course, the above is valid only in some neighbourhood of $;-1;$.






share|cite|improve this answer























  • Why is the first equality possible?
    – pter26
    Nov 27 '18 at 21:54










  • @pter26 That's merely algebra and the Taylor expansion of sine, which is true for any value of $;zinBbb C;$ .
    – DonAntonio
    Nov 27 '18 at 21:55










  • Thanks a lot! So can I tell the residue in $z=-1$ is one from this?
    – pter26
    Nov 27 '18 at 22:02












  • Well, I think it is zero since in the series the first non-constant element goes with $;(z+1)^2;$ , so " there is no coefficient of $;(z+1)^{-1};$" , meaning: its coefficient is zero
    – DonAntonio
    Nov 27 '18 at 22:15














1












1








1






We have



$$frac{sinfracpi2(z+1)}{left[3+(z+1)^2right](z+1)^3}=frac13cdotfrac1{1+left(frac{z+1}{sqrt3}right)^2}cdotfrac1{(z+1)^3}sum_{n=0}^inftyfrac{(-1)^npi^{2n+1}(z+1)^{2n+1}}{(2n+1)!}=$$



$$=frac13left(1-left(frac{z+1}{sqrt3}right)^2+left(frac{z+1}{sqrt3}right)^4-ldotsright)frac1{(z+1)^{color{red}2}}sum_{n=0}^inftyfrac{(-1)^npi^{2n+1}(z+1)^{color{red}{2n}}}{(2n+1)!}$$



and we can see clearly in the above Laurent expansion that the pole at $;z=-1;$ is of order two. Of course, the above is valid only in some neighbourhood of $;-1;$.






share|cite|improve this answer














We have



$$frac{sinfracpi2(z+1)}{left[3+(z+1)^2right](z+1)^3}=frac13cdotfrac1{1+left(frac{z+1}{sqrt3}right)^2}cdotfrac1{(z+1)^3}sum_{n=0}^inftyfrac{(-1)^npi^{2n+1}(z+1)^{2n+1}}{(2n+1)!}=$$



$$=frac13left(1-left(frac{z+1}{sqrt3}right)^2+left(frac{z+1}{sqrt3}right)^4-ldotsright)frac1{(z+1)^{color{red}2}}sum_{n=0}^inftyfrac{(-1)^npi^{2n+1}(z+1)^{color{red}{2n}}}{(2n+1)!}$$



and we can see clearly in the above Laurent expansion that the pole at $;z=-1;$ is of order two. Of course, the above is valid only in some neighbourhood of $;-1;$.







share|cite|improve this answer














share|cite|improve this answer



share|cite|improve this answer








edited Nov 27 '18 at 21:54

























answered Nov 27 '18 at 21:48









DonAntonio

177k1491225




177k1491225












  • Why is the first equality possible?
    – pter26
    Nov 27 '18 at 21:54










  • @pter26 That's merely algebra and the Taylor expansion of sine, which is true for any value of $;zinBbb C;$ .
    – DonAntonio
    Nov 27 '18 at 21:55










  • Thanks a lot! So can I tell the residue in $z=-1$ is one from this?
    – pter26
    Nov 27 '18 at 22:02












  • Well, I think it is zero since in the series the first non-constant element goes with $;(z+1)^2;$ , so " there is no coefficient of $;(z+1)^{-1};$" , meaning: its coefficient is zero
    – DonAntonio
    Nov 27 '18 at 22:15


















  • Why is the first equality possible?
    – pter26
    Nov 27 '18 at 21:54










  • @pter26 That's merely algebra and the Taylor expansion of sine, which is true for any value of $;zinBbb C;$ .
    – DonAntonio
    Nov 27 '18 at 21:55










  • Thanks a lot! So can I tell the residue in $z=-1$ is one from this?
    – pter26
    Nov 27 '18 at 22:02












  • Well, I think it is zero since in the series the first non-constant element goes with $;(z+1)^2;$ , so " there is no coefficient of $;(z+1)^{-1};$" , meaning: its coefficient is zero
    – DonAntonio
    Nov 27 '18 at 22:15
















Why is the first equality possible?
– pter26
Nov 27 '18 at 21:54




Why is the first equality possible?
– pter26
Nov 27 '18 at 21:54












@pter26 That's merely algebra and the Taylor expansion of sine, which is true for any value of $;zinBbb C;$ .
– DonAntonio
Nov 27 '18 at 21:55




@pter26 That's merely algebra and the Taylor expansion of sine, which is true for any value of $;zinBbb C;$ .
– DonAntonio
Nov 27 '18 at 21:55












Thanks a lot! So can I tell the residue in $z=-1$ is one from this?
– pter26
Nov 27 '18 at 22:02






Thanks a lot! So can I tell the residue in $z=-1$ is one from this?
– pter26
Nov 27 '18 at 22:02














Well, I think it is zero since in the series the first non-constant element goes with $;(z+1)^2;$ , so " there is no coefficient of $;(z+1)^{-1};$" , meaning: its coefficient is zero
– DonAntonio
Nov 27 '18 at 22:15




Well, I think it is zero since in the series the first non-constant element goes with $;(z+1)^2;$ , so " there is no coefficient of $;(z+1)^{-1};$" , meaning: its coefficient is zero
– DonAntonio
Nov 27 '18 at 22:15


















draft saved

draft discarded




















































Thanks for contributing an answer to Mathematics Stack Exchange!


  • Please be sure to answer the question. Provide details and share your research!

But avoid



  • Asking for help, clarification, or responding to other answers.

  • Making statements based on opinion; back them up with references or personal experience.


Use MathJax to format equations. MathJax reference.


To learn more, see our tips on writing great answers.





Some of your past answers have not been well-received, and you're in danger of being blocked from answering.


Please pay close attention to the following guidance:


  • Please be sure to answer the question. Provide details and share your research!

But avoid



  • Asking for help, clarification, or responding to other answers.

  • Making statements based on opinion; back them up with references or personal experience.


To learn more, see our tips on writing great answers.




draft saved


draft discarded














StackExchange.ready(
function () {
StackExchange.openid.initPostLogin('.new-post-login', 'https%3a%2f%2fmath.stackexchange.com%2fquestions%2f3016333%2fdetermination-of-the-order-of-a-pole%23new-answer', 'question_page');
}
);

Post as a guest















Required, but never shown





















































Required, but never shown














Required, but never shown












Required, but never shown







Required, but never shown

































Required, but never shown














Required, but never shown












Required, but never shown







Required, but never shown







Popular posts from this blog

Bundesstraße 106

Le Mesnil-Réaume

Ida-Boy-Ed-Garten